You are on page 1of 53

012312424 55689

 9  
problem 1 in chapter 36

m'.(%n2+B $%&'()*+%(,%-)'.(/,+.%01234/+15)'.(/,+.167.8/'93:42;'+(,</3=.('>(:66666??@A@=.3B2:=.(CD':=.3/88:E

-)'.(/,+%D*'o/'p
9 
specific weight
slSQ6tuvQ
w\_xyb

L 
   99 
  !  9   LM4LNO4L

P244 
 44444PP"O"

Q9
 
9ML9!9   L
9Q9 9 9
9
 

! 
9  R zy{|c_}]Zc~\
L024L1L
g


9
9S5QTU
 9L0M 
V
WX
 9
!
99


 RL9
Y Kcc€{‚ƒ
9
R
VWX
 9L
 L
L9R

9


 RL
 9Y „^_‚c‚^‚c|\…
 !9  †9
z‚^‚`
FGH22 1L2   9
FIH2P 1L2 Jb\^‚\x]‡_
4g1201240M4#gP23
FJH23 1L2 5ˆ
FKHPO 1L2 ‰Š{c̀[\x]‡_
4g1201240M4#gP23
Z[\]^_`a\b]c̀]FIHd 5ˆ
‹yxcc\x]‡_
S  041231240M4"2P4"

V
WQ9S5QTU
99 99L 99  9  9 99
9L9
L 
  e tQlŒlŒS6tuS
99  9  9 9  4g1201240M4#gP235
V1
 19  1 9
4310#124034ggMP4
V1
 19  1 9
v6SŽ6586ulS
f Q
9g"3e9 9  L
  h
q 020 5lŽ 99 


9Li
rjk9!9   L
9e99  9  9 9  V1
 19  1 9
L 9   9‘9’
Ž 99 
W
5lŽ Ž   
V1
 19  1 9
L 9   9‘9’
Ž Ž    W
f Q
9g"3e9 9  L
  h
q 4"gQ
 9L
 
99L 9  e 9
!  9 9!9TU
9 5lŽ S 

Q9 9
!
99


 R  9 Rlg"2g V1
 19  1 9
L 9   9‘9’
Ž S 
W
119
 2
1
 19  141 9 9!1234"# 012
012312424 55689
 9  
5(/ ,
 
 
$1
 19  1 9
- 90   91920
$%&'()"2*+ / ,
 
  %
5(/ 3
9
49 9

(  9 

$1
 19  1 9
- 90   91920
/ 3
9 49 9

(  9 
%
,9-
 -
-9'

9


 '-
 9
9 
!
9 9  9 '()"* 56738(89(4(.
.
!: 
  
$.8 84:8%
$1
 19  1 9
- 90
9
1920.

: 
  $.8
84:8%%
. 9()"22
 9
'

-
 -
-9'-2
9 9 
99


 '+
. ;9

$1
 19  1 9
- 90
9
1920. 
;9
%
5478</,.<.(866

119
 2
1
 19  141 9 9!1234"# 212
010213434 55689
 9  
problem 9 in chapter 87

h-)''..(/,(+%%iF*'2j/+'kD $%&'()*+%(,%-)'.(/,+.%01234/+15)'.(/,+.167.8/'93:42;'+(,</3=.('>(:66666?@ABC=.3D2:=.(EF':=.3/88:G

9  ldbf6mnof
pY\qr_
N244  9 O

PQ 


 9 9
9 R 
9O!! 
9 444442s"U0


9 S 9O9
T truv`\wZWx̀Y
 !9  
Mỳyz̀{u|}
HIJN202 9
O
HKJN2"4 ~{[\|`|[|v̀Y
n
HLJNPU2 t|[|{]
HMJNPV4 Iz|v̀Y
L_Y[|YqZ€\
WXYZ[\]^Y_Z̀]ZHLJa 4010V1340#004P3#

‚{ƒù]XYqZ€\
b  4010V1340#004P3#

„rqỳỲqZ€\
031"01340V42"U34

mfd…†d…b6mnb
o6b‡6586ndb
edd  9 

ˆ1
 19  1 9
S 9‰   9Š9R
d  9 

ed9


b 999   9 Oc99SS9 9 9 9
99

9S  ˆ1
 19  1 9
dU"20 S 9‰   9Š9R
9


87.51

edm9 o   9
ˆ1
 19  1 9
87.1 S 9‰   9Š9R
e f
9U"0c m9 o   9‹
5d9

f9 

e bO9
ˆ1
 19  1 9
f9U"gc 9 
    9 9 S 9‰   9Š9R
87.B

119
 3
1
 19  141 9 9!13"22# 013
010213434 55689
 9  
$%& '()*+,-./0 99

69 


: ;<9=
1233 0""#3
5>>  9 

1014# 9 9 ?1
 19  1 9
7 9@   9A9B
4233 05431 >  9 
=
CDEF8>GH>I>;
>  99 >  
?1
 19  1 9
7 9@
9
A9B@> 
69 97 98 >  =
J
 >  99 
5
 9
?1
 19  1 9
7 9@
9
A9B@J

>  99 5
 9=
 
  K

>  99 5  9


?1
 19  1 9
7 9@
9
A9B@
K
9>  99
5  9=
5IEGLM6;L;>G6D

119
 3
1
 19  141 9 9!13"22# 313
012312424 55689
 9  
problem 2 in chapter
20

`)0*'a4-C &'()*+,-'*.'/+)0*1.-0'234561-37+)0*1.-03890:1);5<64=)-*.>15?0*)@*<88888AAABA?05C4<?0*DE)<?051::<F

/+)0*1.-'E,)b1)c
9  TeVR6fghR
ijklmn
2 9 
000G2
9
 9

  HH9H#I04 J3! 
9
G 
9H 44444"""0"
2J4 13 ompqrkstur\j
K
LM
 9
9N9 9  
  O 
vrwwrxypz{
KLM
 9 P 
9
G 
9O 9
P
KL6H
2   J"H  HH 999  
9 Q!
 99  H |y}kzrz}zrqj~
P 
9O 9
oz}zy^
KLM
 99   
9O   9
K9LR9 9 9 9 9
H9J"  "4P Q
99  99  9H 
34P  H9 €nj}zjltk
M
 9 
  P
9

 P! 99N999 99   H9O 0412_1240%4J2#2$

KHLR9 P 
 9 99 !
9 H!  
 9
H  ƒy„pr^Zjltk
6
9H! 0412_1240%4J2#2$

669


  …mlrwrjltk
6669 
H! 0014%1240%4$3%"J

6S H
9H! fRe†Se†V6fgV
S 9 H! 4010%1240%040$0J5
T99H!  H 
  H 
KL KUL K1
 19  1 9
V 9
 
9 9 9 
3  
#4G2!
9 9
9
H! 0412_1240%4J2#2$5
W X K1
 19  1 9
YZ[ Y\d]^[ 4_10$1240_433%#2
K1
 19  1 9
4 0$
h6V‡6586geV
0 0J 5e‡ M
9
2 #% †9 9

e  9 

3 _4 K1
 19  1 9
H 9ˆ   9‰9Nˆ
# 04% ‡ M
9 †9 9

e  9 
L
" %"
ŠgfM8eh‹e†eV
J "% P P
$ 3% K1
 19  1 9
H 9ˆ
9
‰9NˆP
119
 2
1
 19  141 9 9!13"#$2 01%
012312424 55689
 9  
, - 5KLMNOIENEGM6P
&.( &/J01(
% 22
3 24
04 0%
00 0$
02 02
456
 9 7 
9
8 
99  9
9  999  3 :
456
 99
8 
99 
2   "9  99  !
9 9:
4 56
 99
8 
99 
"   "9  99  !
9 9:
4;56
 99  97 
99 
"   "9  99  !
9 9:
 !9 
&'(0#
&)(22
&*(32
&+(#4
<.=>?@1A=B>C1>&*(D
E 
4
5F G2420H

45I9 7 
 
9 99
8 
9 979
9
9 9  
  

45I99  97 
99 
2"  99  97 7 9 
7 
972"F 9 7 
H

F 92" H

119
 2
1
 19  141 9 9!13"#$2 21%
012312424 55689
 9  

&'(99   
9 99  9) ) 9 ) 
 
9)*"

&9'+ ,2424-

&.'(9 ) 
 9 99 
 
9 
 9
9/9.   !
9  
9.!-! 

99

9,


   9.9 !
9 
  9 99
9 9
-9 9. 9- 
9
 9 9 9
 
9(9 ) 
 99 
.!- .
9.!-

9 .!
&'(9

  .. 9(9

9
9.!. 9 
.!-99
 99
0
!
 1 
 9. 9
.9   9.
  6 9
9-
9
9.!
. 99 
.!

2345 546377 853469:;<=>5 ?457<@>:;<=>5


4 0$ 0* 0 432
0 0* 0* 4 4
2 #% 0* 32 040A
3 A4 0* $# 23"*
# 04% 0* A2 2A2A

119
 2
1
 19  141 9 9!13"#$2 31%
012312424 55689
 9  
&'() )(*'++ ,)'(*-./012) 3()+042./012)
" %" 05 56 206$
5 "% 05 #2 033$
$ 3% 05 22 $44
% 25 05 04 30%
6 24 05 # 02$
04 0% 05 2 45#
00 0$ 05 0 432
02 05 05 4 4

 3#671
89
9
9 9  
    9 9:2420

899;  ;; 9


9
 9 99

99 
 9< 9


9 9 ; 9

 =9>! ?;9
 
9 0

89 9 
9
@ 
9 
119
 2
1
 19  141 9 9!13"#$2 #1%
012312424 55689
 9  

&'(9 9
9!9)    * 
) 
 9  

)9 
* 
+  

9!9 9)9 * 

  ,9
  9
 9
6 

9,

9  * 
9

9*-F.3(9 ,
9 
9) 
* 
)

  (9 
99 9 9*/.299 * 

 
9
0123 2456708931:3;<07=707>613? >@A1497>613? 616;B 2456708931:3;<07C707>613?
4 432 4 432 40D
0 4 4 4 4
2 040E 4 040E "4E
3 23"D 432 23%% 00E#
# 2E2E 4 2E2E 0#D"
" 20E$ 040E 320D 0D4%
D 033$ 23"D 3DE3 0%#$
$ $44 2E2E 3D2E 0%0"
% 30% 20E$ 2"0" 02"%
E 02$ 033$ 0#D" $32
04 4D# $44 $D# 3%2
00 432 30% 3"4 0$"
02 4 02$ 02$ 4D#
03 4 4D# 4D# 432
0# 4 432 432 40D
0" 4 4 4 4

119
 2
1
 19  141 9 9!13"#$2 "1%
012312424 55689
 9  

'99
( 
9) 
"  

9) 99
( * 
 
9

+ ,6)
   
 9)9 * 

  -9./ 9999
  
* 
) 
 6 9
9-
9 
9)) 93 * 
-9

))9*3- 9
./ 9
45052678 >?@16: 8073: B12380 B7B80 818<C5DEF
0123 09:31;3<=0 >813A >813A >813A >813A @23G?H
4 432 4 4 4 4 432
0 4 4 4 4 4 4
2 040I 4 4 4 4 040I
3 23"% 432 4 4 4 23&&
# 2I2I 4 4 4 4 2I2I
" 20I$ 040I 4 4 4 320%
% 033$ 23"% 432 4 4 3$2"
$ $44 2I2I 4 4 4 3%2I
& 30& 20I$ 040I 4 4 3"33
I 02$ 033$ 23"% 432 4 3&"2
04 4%# $44 2I2I 4 4 3%I3
00 432 30& 20I$ 040I 4 3"%%
02 4 02$ 033$ 23"% 432 3&"2
119
 2
1
 19  141 9 9!13"#$2 %1&
012312424 55689
 9  
*+&+(,-. 456',0 .&-)0 8'().& 8-8.& .'.29+:;<
&'() &/0)'1)23& 4.')7 4.')7 4.')7 4.')7 6()=5>
03 4 4?# $44 2@2@ 4 3?@3
0# 4 432 30% 20@$ 040@ 3"??
0" 4 4 02$ 033$ 23"? 3%24

A9 
 B 
9C"D9EF 999AA
9
C
9G6

C
9
9 C9!99 3"
31
3@31HI9" C 
 

9C

 
9 EF 9
 9A C
"DA  9 AA9 9 9D

 

 A3"GI9 9
 9
9
9 9A99

 B 
9
 9A99H
&'() ;<6()=5 9211-,1+;<6()=5 J+&+(,-.+&/0)'1)23&
K4 432 4 40@
K0 4 4 4
K2 040@ 4 ?00
K3 23%% 4 0#32
K# 2@2@ 4 0$"$
K" 320? 432 0@04
K? 3$2" 4 223"
K$ 3?2@ 040@ 0"??
K% 3"33 23%% ?%%
K@ 3%"2 2@2@ ""#
04 3?@3 320? 2%$
00 3"?? 3$2" L4@?

119
 2
1
 19  141 9 9!13"#$2 $1%
012312424 55689
 9  
&'() *+,()-. /0112314*+,()-. 54&4(3264&78)'1)09&
02 3%"2 3:2; 03#
03 3:;3 3"33 4;:
0# 3":: 3%"2 <0$2 0.76

0" 3%24 3:;3 4;:



 0000$31=

>99
? 
9@ 
"  

9@ 99
? A 
 
9

BCD>99  @9 9 9@ 9" A 




E 
" F

119
 2
1
 19  141 9 9!13"#$2 %1%
012314343 55689
 9  
problem 4 in
chapter 45

z)0*'{4-E &'()*+,-'*.'/+)0*1.-0'234561-37+)0*1.-03890:1);5<64=)-*.>15?0*)@*<88888ABCDB?05E4<?0*FG)<?051::<H

/+)0*1.-'G,)|1)}
9  ~fU6€XU
‚]`ƒ„c
 9


O!  9   P4PQ4PQ4PR"3Q"3Q"3S  9 T94 33333V2%$2
P1 
4
9R0#O5
SU999 
99  

O

 O 9P34V R"2S

…„†‡d̀ˆ^[d‰]
Q 9 9P2"O R4V3W5
SX99  9 9 
  P
 
9 9 
99Y9 
P
 P
P9Q  9
Q NdŠŠd‹Œ†Ž
 !9  9
Q
Œ_`d_d‡]
IJK20 ‘9
ILK2Z …_Œa
J‹d‡]
IMK#4 Mc]_]ƒ^’`
INK"2 0013Z1430%044044
W
[\]^_`ab]c^da^INKe “Œ”†da\]ƒ^’`
0013Z1430%044044
f  W
•„ƒdŠd]ƒ^’`
ghijklmnopqrsrpskthjukv 3010Z1434300V$32
U9 9   P9P
9
9 5W
€U–—–f6€Xf
0310%1430%3#VZ045
R1
 19  1 9
0314$1430%3"4V2V5
U9
 P9 9 9     R1
 19  1 9
031201430%3VVZ2#5
45.7 R1
 19  1 9
U9
9   9 9
9 w 99x U
9V4ZY9
y  9 
0013"1430%303"VV
R1
 19  1 9
0013Z1430%044044
R1
 19  1 9
U9P
 P
P9Q  6f˜6586Xf
5W9

W

9  
W
9

R1
 19  1 9
P 9™   9š9y™
W9

W
 9
9  
W
9
S
119
 4
1
 19  131 9 9!12"#$% 012
012314343 55689
 9  
&'(&)*+,-).
/9 9   090
9
91232"3
4232"3 5<= > 

?1
 19  1 9
/9
 09 9 9     0 93   9@9:3
= > 
A
45.7
5<= =   
/9
9   9 9
9 5 996 /
974899
:  9  ?1
 19  1 9
0 93   9@9:3
= =    A
5<= B99 

?1
 19  1 9
0 93   9@9:3
= B99 
A
5<= /
 
 
?1
 19  1 9
0 93   9@9:3
= /
 
  A
/90
 0
09; 
5<= C
9
D9 9

<  9 

?1
 19  1 9
0 93   9@9:3
= C
9 D9 9

<  9 
A
EFGC8<HB<D<>
I
9
5 9  9
?1
 19  1 9
0 93
9
@9:3I

5 9  9A
I
9


9

 9  9
?1
 19  1 9
0 93
9
@9:3I



9
 9  9
I
9

?1
 19  1 9
0 93
9
@9:3I

A
5DGHJ=/>J><H6F
5<=</DK
5<=<B/K
5<=<>/K
5<=<=FK
5<=<C<K

119
 4
1
 19  131 9 9!12"#$% 412
Problem 6 in chapter 35

 Return to Questions (/admin/questions/0?sfield=magento_id&stext=0000055627&sdka=&stype=&sdiff=)

Test Bank Question


preview

QUESTION DATA
Question
Vendor
For the soil in Prob. 5, what volume of water must be added to obtain 1 yd3 of soil at the maximum density if 0000055627
the soil is originally at 10% water content (dry basis)? Solving Time

Answers Difficulty
(A) 6 gal easy
Quantitative?
(B) 14 gal
Yes
(C) 18 gal Status
Active
(D) 31 gal
Created On
11/13/2018 05:28:57 PM
The answer is (D).
Published On
11/13/2018 05:28:57 PM
Solution Modified On
At maximum density, the masses of water and solids in 1 ft3 of soil are 05/27/2020 03:07:47 AM
OTHER VERSIONS
m s = 107.5 lbm
01/19/2018 11:04:44 PM
m w = (0.19) (107.5 lbm) = 20.43 lbm
(/admin/questions/previ
At 10% water content, γd ≈ 89.1 lbf/ft3. For 1 ft3,
11/06/2018 10:38:11 PM
m s = 89.1 lbm
(/admin/questions/previ
m w = (0.10) (89.1 lbm) = 8.91 lbm

To get 1 yd3 (27 ft3) of maximum density soil, the volume of 10% moisture soil is 11/12/2018 09:10:50 PM
(/admin/questions/previ
3
ft
(27 
3
) (107.5 lbm)
3 3
DISCIPLINES
yd 32.58 ft /yd  of
=
89.1 lbm 10% moisture soil PE Civil: Construction
(/admin/questions/index
The required water for 1 yd3 of soil is sfield=discipline&stext=
3
Civil: Construction)
⎛ ft lbm ⎞
(27  ) (20.43  )
3 3
⎜ yd ft ⎟ gal
3
(1 yd ) ⎜ ⎟ (7.48  ) PE Civil: Geotechnical
⎜ 3
⎟ 3
⎜ ft lbm ⎟ ft (/admin/questions/index
− (32.58  ) (8.91  )

yd ft
3 ⎠ sfield=discipline&stext=
lbm
Civil: Geotechnical)
62.4 
3
ft
PE Civil: Transportation
= 31.33 gal (31 gal)
(/admin/questions/index
sfield=discipline&stext=
Civil: Transportation)
problem 2 in chapter 68

 Return to Questions (/admin/questions/0?sfield=magento_id&stext=0000056043&sdka=&stype=&sdiff=)

Test Bank Question


preview

QUESTION DATA
Question
Vendor
A 25 ft high, 10 in CMU wall is partially grouted and reinforced. The as-manufactured face-shell thickness is 0000056043
1.375 in. The wall is non-load-bearing and is subjected to wind only. The wall is simply supported at top and Solving Time
bottom, and reinforcement is placed in the center of the wall. Use fm and n .
' 2
= 2000 lbf/in = 16.1

Difficulty
easy
Scenario Quantitative?
Yes
(a) With reinforcement spaced at 24 in, determine the largest reinforcing bars that can be used such that the
Status
neutral axis falls within the face shell of the unit.
Active
Created On
Answers
11/08/2018 07:42:04 PM
(A) no. 4 bars Published On
11/08/2018 07:42:04 PM
(B) no. 5 bars
Modified On
(C) no. 6 bars 05/27/2020 03:08:17 AM
OTHER VERSIONS
(D) no. 7 bars
03/21/2018 07:36:08 PM
The answer is B. (/admin/questions/previ

Solution 08/13/2018 02:19:35 PM


(a) 10 in is the nominal thickness. The actual thickness is 10 in − 0.375 in = 9.625 in. (/admin/questions/previ

kd = tfs = 1.375 in
11/08/2018 07:42:05 PM
9.625 in
d = = 4.81 in (/admin/questions/previ
2

tfs 1.375 in DISCIPLINES


k = = = 0.286
d 4.81 in
PE Civil: Structural
From Eq. 68.4, (/admin/questions/index
sfield=discipline&stext=
2
k + 2ρnk − 2ρn = 0 Civil: Structural)
2
(0.286) + 2ρ (16.1) (0.286) − 2ρ (16.1) = 0
KNOWLEDGE AREAS
ρ = 0.00356

Component design and


Use b = 24 in, the vertical bar spacing. detailing
(/admin/questions/index
A s = ρbd
sfield=area&stext=Com
= (0.00356) (24 in) (4.81 in)
design and detailing)
2
= 0.411 in [for every 24 in]
PRODUCTS USED IN
The area of a no. 6 bar is 0.44 in2, which is too large.

2
The area of a no. 5 bar is 0.31 in , which is acceptable.
Scenario
For parts (b) through (f), assume the wall has no. 6 vertical bars spaced 32 in apart.

(b) Determine the location of the neutral axis measured from the compression face.

Answers
(A) 1.0 in

(B) 1.3 in

(C) 1.7 in

(D) 2.0 in

The answer is B.

Solution
(b) Work on a per-foot-of-wall basis. From Table 68.3,

2
A s = 0.166 in /ft

On a per-foot basis, b = 12 in.

2
As 0.166 in
ρ = = = 0.00288
bd (12 in) (4.81 in)

ρn = (0.00288) (16.1) = 0.0463

From Eq. 68.4,


−−−−−−−−−−
2
k = √ 2ρn + (ρn) − ρn

−−−−−−−−−−−−−−−−−−−
2
= √ (2) (0.0463) + (0.0463) − 0.0463

= 0.262

kd = (0.262) (4.81 in) = 1.26 in (1.3 in)

Scenario
(c) Determine the resisting moment assuming steel governs.

Answers
(A) 17,000 in-lbf/ft

(B) 20,000 in-lbf/ft

(C) 23,000 in-lbf/ft

(D) 24,000 in-lbf/ft

The answer is C.

Solution
(c) Use Eq. 68.5 and Eq. 68.7.
2
Fs = 32,000 lbf/in

k 0.262
j = 1 − = 1 − = 0.913
3 3

Ms = A s Fs jd

2
in lbf
= (0.166 ) (32,000 ) (0.913) (4.81 in)
2
ft in

= 23,328 in-lbf/ft (23,000 in-lbf/ft)

Scenario
(d) Determine the allowable bending stress.

Answers
(A) 670 lbf/in2

(B) 890 lbf/in2

(C) 1000 lbf/in2

(D) 1100 lbf/in2

The answer is A.

Solution
(d) Use Eq. 68.22.

1 '
1 lbf
Fb = fm = ( ) (2000 )
3 2
3 in

2 2
= 667 lbf/in (670 lbf/in )

Scenario
(e) Determine the resisting moment of the wall.

Answers
(A) 17,000 in-lbf/ft

(B) 22,000 in-lbf/ft

(C) 23,000 in-lbf/ft

(D) 29,000 in-lbf/ft

The answer is A.

Solution
(e) Use Eq. 68.6 and Eq. 68.8.
2
jk
Mm = Fb bd ( )
2

lbf in 2
(0.913) (0.262)
= (667 ) (12 ) (4.81 in) ( )
2
in ft 2

= 22,148 in-lbf/ft

MR = lesser of Ms and Mm

= 17,000 in-lbf/ft

Therefore, steel governs.

Scenario
(f) Determine the shear capacity.

Answers
(A) 1100 lbf/ft

(B) 3400 lbf/ft

(C) 3900 lbf/ft

(D) 4300 lbf/ft

The answer is A.

Solution
(f) Work on a per-foot-of-wall basis. Using Eq. 68.2,
−−−−−− −

−−
'
lbf
Fv = √ fm = √ 2000 
2
in
2 2
= 44.7 lbf/in < 50 lbf/in [OK]

Determine the core width for a 12 in stretcher block. Since there are two cells per core,

15.625 in − 1.5 in
b = = 7.1 in
2

For partially grouted walls, cells are grouted at vertical rebars. Bar spacing is 32 in on center. Therefore, the
cell width is

7.1 in in
bw = ( ) (12  ) = 2.65 in/ft
32 in ft

Use Eq. 68.16.

VR = Fv (btfs + bw (d − tfs ))

lbf in
  = (44.7  ) ((12  ) (1.375 in)
2
in ft

in
  + (2.65  ) (4.81 in − 1.375 in))
ft

= 1144 lbf/ft (1100 lbf/ft)

Scenario
(g) Determine the area of steel required to produce a balanced design condition.
Answers
(A) 0.15 in2/ft

(B) 0.18 in2/ft

(C) 0.21 in2/ft

(D) 0.25 in2/ft

The answer is A.

Solution
(g) From Eq. 68.10,

nFb
ρ bal =
Fs
2Fs (n + )
Fb

lbf
(16.1) (667 )
2
in
=
lbf
⎛ 32,000 ⎞
2
lbf ⎜ in ⎟
(2) (32,000 ) ⎜ 16.1 + ⎟
2 ⎜ lbf ⎟
in
⎝ 667 ⎠
2
in

= 0.00262

in
A s,bal = ρ bd = (0.00262) (12 ) (4.81 in)
bal
ft

2 2
= 0.151 in /ft (0.15 in /ft)

Scenario
(h) Determine the maximum moment on the wall if it is subjected to a 25 lbf/ft2 wind load.

Answers
(A) 23,400 in-lbf/ft

(B) 24,200 in-lbf/ft

(C) 24,800 in-lbf/ft

(D) 25,100 in-lbf/ft

The answer is A.

Solution
(h) The maximum moment of the wall is

lbf 2
in
(25 ) (25 ft) (12 )
2 2
wl ft ft
M = =
8 8

= 23,438 in-lbf/ft (23,400 in-lbf/ft)

Scenario
(i) Using ρ = 0.004 and d = 4.81 in, and assuming the wall is fully grouted determine the wind-induced
service moment the wall can withstand.

Answers
(A) 24,000 in-lbf/ft

(B) 32,000 in-lbf/ft

(C) 38,000 in-lbf/ft

(D) 47,000 in-lbf/ft

The answer is A.

Solution
(i) The wind-induced service moment is calculated as follows.

ρn = (0.004) (16.1) = 0.0644

−−−−−−−−−−
2
k = √ 2ρn + (ρn) − ρn

−−−−−−−−−−−−−−−−−−−
2
= √ (2) (0.0644) + (0.0644) − 0.0644

= 0.300

k 0.300
j = 1 − = 1 − = 0.900
3 3

in
A s = ρbd = (0.004) (12 ) (4.81 in)
ft

2
= 0.231 in /ft

From Eq. 68.7,

Ms = A s Fs jd

2
in lbf
= (0.231 ) (24,000 ) (0.900) (4.81 in)
2
ft in

= 24,000 in-lbf/ft

Scenario
(j) Assume the wall is fully grouted masonry and that the wall must resist a moment of 20,000 in-lbf/ft. With
no. 5 bars spaced at 16 in within a 10 in thick wall, determine the steel stress.

Answers
(A) 14,000 lbf/in2

(B) 16,000 lbf/in2

(C) 18,000 lbf/in2

(D) 20,000 lbf/in2

The answer is D.

Solution
(j) From Table 68.3,
2
A s = 0.230 in /ft

2
in
0.230
As ft
ρ = =
bd in
(12 ) (4.81 in)
ft

= 0.00398

ρn = (0.00398) (16.1) = 0.0641

−−−−−−−−−−
2
k = √ 2ρn + (ρn) − ρn

−−−−−−−−−−−−−−−−−−−
2
= √ (2) (0.0641) + (0.0641) − 0.0641

= 0.300

k 0.300
j = 1 − = 1 − = 0.900
3 3

in-lbf
20,000
M ft
fs = =
2
A s jd in
(0.230 ) (0.900) (4.81 in)
ft

2 2
= 20,087 lbf/in (20,000 lbf/in )
problem 3 in chapter 68

 Return to Questions (/admin/questions/0?sfield=magento_id&stext=0000056042&sdka=&stype=&sdiff=)

Test Bank Question


preview

QUESTION DATA
Question
Vendor
An 8 in hollow, unreinforced, ungrouted concrete masonry wall, 12 ft high, is simply supported at top and 0000056042
bottom. Type M mortar-cement is used. For parts (a) through (e), fm
'
= 1800 lbf/in
2
. Solving Time

Difficulty
Scenario easy
Quantitative?
(a) Determine the allowable stresses for permanent loads.
Yes
Status
Answers Active
(A) Fa = 320 lbf/in2, Fb = 800 lbf/in2 Created On
11/08/2018 07:42:05 PM
(B) Fa = 390 lbf/in2, Fb = 600 lbf/in2
Published On
(C) Fa = 390 lbf/in2, Fb = 800 lbf/in2 11/08/2018 07:42:05 PM
Modified On
(D) Fa = 520 lbf/in2, Fb = 600 lbf/in2
05/27/2020 03:08:17 AM
OTHER VERSIONS
The answer is B.
03/21/2018 07:36:08 PM
Solution (/admin/questions/previ

(a) From App. 68.A, for an 8 in, single wythe, ungrouted wall,
10/29/2018 06:26:45 PM
r = 2.84 in (/admin/questions/previ
3
Sx = 81 in /ft

A n = 30 in /ft
2
11/08/2018 07:42:05 PM
4 (/admin/questions/previ
Ix = 334 in /ft

in DISCIPLINES
(12 ft) (12 )
h ft
= = 50.7 PE Civil: Structural
r 2.84 in
(/admin/questions/index
Since 50.7 is less than 99, use Eq. 68.20. sfield=discipline&stext=
Civil: Structural)
2

1 '
h
Fa = fm (1 − ( ) ) KNOWLEDGE AREAS
4
140r

2
Component design and
1 lbf 50.7
= ( ) (1800 ) (1 − ( ) )
detailing
2
4 in 140 (/admin/questions/index
2 2
sfield=area&stext=Com
= 391 lbf/in (390 lbf/in )
design and detailing)
1 '
1 lbf
Fb =
3
fm = ( ) (1800
2
) PRODUCTS USED IN
3 in

2
= 600 lbf/in
Scenario
(b) Determine the maximum vertical load the wall can be designed for with a 2 in eccentricity.

Answers
(A) 6700 lbf/ft

(B) 7900 lbf/ft

(C) 10,000 lbf/ft

(D) 14,000 lbf/ft

The answer is B.

Solution
(b) The axial and bending stresses on the wall are

P P
fa = = = 0.033P
2
An in
30
ft

M Pe P (2 in)
fb = = = = 0.025P
3
Sx Sx in
81
ft

From Eq. 68.17,

fa fb
+ ≤ 1
Fa Fb

0.033P 0.025P
+ = 1
lbf lbf
391 600
2 2
in in

P = 7932 lbf/ft

Check buckling criteria.

'
lbf 2
Em = 900fm = (900) (1800 ) = 1,620,000 lbf/in
2
in

2 3
1 1
π Em I e
P ≤ Pe = ( ) (1 − 0.577 ( ))
4 4 2
h r

4
⎛ lbf in ⎞
2
π (1,620,000 ) (334 )
2
⎜ 1 in ft ⎟
= ( )⎜ ⎟
⎜ 2 ⎟
4 ⎜ in ⎟
2
(12 ft) (12 )
⎝ ⎠
ft

3
2 in
×(1 − (0.577) ( ))
2.84 in

= 13,471 lbf/ft

Buckling does not control. Therefore,

P max = 7932 lbf/ft (7900 lbf/ft)

Scenario
(c) Determine the maximum wind load if the wall carries no vertical load. Neglect the self-weight of the wall.
Answers
(A) 9.4 lbf/ft2

(B) 12 lbf/ft2

(C) 16 lbf/ft2

(D) 18 lbf/ft2

The answer is A.

Solution
(c) With a wind load, w, in lbf/ft2, the moment resisted by the wall is

2
in
w((12 ft) (12 ))
2
wh ft
M = =
8 in
(8) (12 )
ft

  = 216w [in in-lbf/ft]

The bending stress is

M 216w
2
fb = = = 2.67w [in lbf/in ]
3
Sx in
81
ft

(2.67w has units of lbf/in2 when w has units of lbf/ft2.)

From Table 68.1, the allowable stress is

2
Fb = 25 lbf/in

Set fb equal to Fb.

lbf
2.67w = 25
2
in

2 2
w = 9.36 lbf/ft (9.4 lbf/ft )

Scenario
For parts (d) and (e), the wall is subjected to an axial load of 5000 lbf/ft. There is no lateral load.

(d) Determine the allowable in-plane shear stress.

Answers
(A) 64 lbf/in2

(B) 110 lbf/in2

(C) 120 lbf/in2

(D) 130 lbf/in2

The answer is A.

Solution
(d) Fv is the least of the following [ACI 530 Sec. 8.2.6.2].
−−−−−−−

−− lbf
' 2
1.5√ f = 1.5√ 1800 = 63.6 lbf/in
m
2
in

lbf
(0.45) (5000 )
lbf Nv lbf ft
37  + 0.45 ( ) = 37 +
2 2 2
in An in in
30
ft
2
= 112 lbf/in

120 lbf/in2

The first criterion controls.

2 2
Fv   = 63.6 lbf/in (64 lbf/in )

Scenario
(e) Assuming the wall has no net tension, determine the maximum in-plane shear force the wall can be
designed for.

Answers
(A) 900 lbf/ft

(B) 1000 lbf/ft

(C) 1100 lbf/ft

(D) 1300 lbf/ft

The answer is D.

Solution
(e) From Eq. 68.37,

3V
fv =
2A n

Set fv equal to Fv.

2A n Fv
V =
3
2
in lbf
(2) (30 ) (63.6 )
2
ft in
=
3

= 1272 lbf/ft (1300 lbf/ft)

Scenario
(f) Determine the minimum net area required to sustain an 18,000 lbf/ft axial load given that
.
' 2
fm = 1800 lbf/in

Answers
(A) 30 in2/ft

(B) 39 in2/ft
(C) 46 in2/ft

(D) 52 in2/ft

The answer is C.

Solution
(f) From part (a), since buckling does not control, Fa = 391 lbf/in2.

P
Fa =
An

lbf
18,000
lbf ft
391 =
2
in An

2
An = 46 in /ft

Scenario
For parts (g) through (j), assume the wall is subjected to a vertical dead load, D, of 4500 lbf/ft (e = 2.5 in)
and a lateral wind load, w, of 35 lbf/ft2.

(g) Considering both wind and dead loads, determine the flexural compressive stress in the masonry.

Answers
(A) 120 lbf/in2

(B) 140 lbf/in2

(C) 160 lbf/in2

(D) 180 lbf/in2

The answer is C.

Solution
(g) At the mid-height of the wall,

2
Pe wh
M = +
2 8

lbf
(4500 ) (2.5 in)
ft
=
2
2
lbf in
(35 ) ((12 ft) (12 ))
2
ft ft
+
in
(8) (12 )
ft

= 13,185 in-lbf/ft

in-lbf
13,185
M ft
fb = =
3
Sx in
81
ft

2 2
= 163 lbf/in (160 lbf/in )
Scenario
(h) Considering both wind and dead loads, determine the masonry strength, fm
'
, required.

Answers
(A) 890 lbf/in2

(B) 950 lbf/in2

(C) 1200 lbf/in2

(D) 1500 lbf/in2

The answer is C.

Solution
(h) The allowable axial stress is

1 '
h
Fa = fm (1 − ( ) )
4
140r

1 '
50.7
= fm (1 − ( ) )
4
140

'
= 0.218fm

lbf
4500
P ft 2
fa = = = 150 lbf/in
2
An in
30
ft

The allowable bending stress is

'
Fb = 0.334fm

Use the interaction equation, Eq. 68.17.

fa fb
+ = 1
Fa Fb

lbf lbf
150 163
2 2
in in
+ = 1
' '
0.218fm 0.334fm

' 2 2
fm = 1176 lbf/in (1200 lbf/in )

Scenario
(i) Determine the flexural tension capacity required.

Answers
(A) 6 lbf/in2

(B) 10 lbf/in2

(C) 13 lbf/in2

(D) 15 lbf/in2

The answer is C.
Solution
(i) For dead and wind loads,

−fa + fb = Ft

lbf lbf
−150 + 163 = Ft
2 2
in in
2
Ft = 13 lbf/in

Considering dead load only, the moment at the top of the wall is

lbf
P e = (4500 ) (2.5 in)
ft

= 11,250 in-lbf/ft

in-lbf
11,250
M ft
fb = =
3
Sx in
81
ft
2
= 139 lbf/in

−f + f = Ft
a b

lbf lbf lbf


−150 + 139 = −11 = Ft
2 2 2
in in in

There is no tension in the wall. The dead and wind load case controls.

2
F t,reg = 13 lbf/in

Scenario
(j) Determine the minimum required masonry strength, fm
'
.

Answers
(A) 890 lbf/in2

(B) 950 lbf/in2

(C) 1100 lbf/in2

(D) 1500 lbf/in2

The answer is C.

Solution
(j) For dead and wind loads, from part (h), fm
'
= 1176 lbf/in2.

Check dead load only. From part (h),


2
fa = 150 lbf/in

'
Fa = 0.218fm

From part (i),


2
fb = 139 lbf/in

1 '
Fb = fm
3

fa fb
+ = 1
Fa Fb

lbf lbf
150 139
2 2
in in
+ = 1
' '
0.218fm 0.333fm

' 2 2
fm = 1105 lbf/in (1100 lbf/in ) [governs]
problem 1 chapter 49

 Return to Questions (/admin/questions/0?sfield=magento_id&stext=0000055792&sdka=&stype=&sdiff=)

Test Bank Question


preview

QUESTION DATA
Question
Vendor
(Time limit: one hour) The following information is submitted for a proposed concrete mix. 0000055792
Solving Time
cement
Difficulty
 specific gravity 3.15 easy
Quantitative?
fine aggregate
Yes
Status
 fineness modulus 2.65
Active
 specific gravity 2.48 Created On
11/08/2018 07:40:57 PM
 absorption 3.0% Published On
11/08/2018 07:40:57 PM
coarse aggregate Modified On
05/27/2020 03:08:01 AM
 specific gravity 2.68 OTHER VERSIONS

 dry bulk density 105 lbf/ft³ 03/21/2018 07:35:48 PM


(/admin/questions/previ
 absorption 0.7%

10/29/2018 06:25:58 PM
concrete
(/admin/questions/previ
 slump 5 in
11/08/2018 07:40:57 PM
 water 5.0 gal/sack (/admin/questions/previ

 air content 4% DISCIPLINES

 cement content 6.5 sack mix PE Civil: Structural


(/admin/questions/index
 coarse aggregate 0.57 ft³/ft³ bulk sfield=discipline&stext=
Civil: Structural)

Scenario PE Civil: Construction


(/admin/questions/index
(a) What is the dry weight of the coarse aggregate in 1 yd3 of concrete? sfield=discipline&stext=
Civil: Construction)
Answers
(A) 1120 lbf/yd3 PE Civil: Geotechnical
(B) 1300 lbf/yd3 (/admin/questions/index
sfield=discipline&stext=
(C) 1500 lbf/yd3 Civil: Geotechnical)
(D) 1600 lbf/yd3
PE Civil: Transportation
The answer is D. (/admin/questions/index
sfield=discipline&stext=
Solution Civil: Transportation)

(a) The weight of the coarse dry aggregate is


PE Civil: Water
3 3
ft lbf ft Resources and
(0.57  ) (105  ) (27  )
ft
3
ft
3
yd
3 Environmental
3 3
(/admin/questions/index
= 1616 lbf/yd (1600 lbf/yd )
sfield=discipline&stext=
Civil: Water Resources
and Environmental)
Scenario
KNOWLEDGE AREAS
(b) What is the absolute volume of the coarse aggregate in 1 yd3 of concrete?
Materials and material
Answers properties
(/admin/questions/index
(A) 8.2 ft3/yd3
sfield=area&stext=Mate
(B) 8.3 ft3/yd3 and material properties

(C) 8.7 ft3/yd3


Materials
(D) 9.7 ft3/yd3 (/admin/questions/index
sfield=area&stext=Mate
The answer is D.
)

Solution PRODUCTS USED IN

(b) The volume of coarse aggregate is

lbf
1616 
3
yd
3 3 3 3
= 9.66 ft /yd (9.7 ft /yd )
lbf
(2.68) (62.4  )
3
ft

Scenario
(c) What is the water-cement ratio?

Answers
(A) 0.42

(B) 0.44

(C) 0.46

(D) 0.48

The answer is B.

Solution
(c) The water-cement ratio is
gal lbf
(5  ) (8.34  )
sack gal
= 0.44
lbf
94 
sack

Scenario
(d) What is the weight of cement in 1 yd3 of fresh concrete?

Answers
(A) 610 lbf/yd3

(B) 720 lbf/yd3

(C) 820 lbf/yd3

(D) 910 lbf/yd3

The answer is A.

Solution
(d) The weight of cement in 1 yd3 is

sacks lbf
3 3
(6.5  ) (94  ) = 611 lbf/yd (610 lbf/yd )
3
yd sack

Scenario
(e) What is the absolute volume of fine aggregate in 1 yd3 of fresh concrete?

Answers
(A) 7.8 ft3

(B) 8.8 ft3

(C) 9.8 ft3

(D) 11 ft3

The answer is B.

Solution
(e) To find the volume of fine aggregate, the volume of all other components must be found.

ingredient weight (lbf) volume (ft³)

cement 611
611 lbf
= 3.11
lbf
(3.15) (62.4  )
3
ft

coarse aggregate 1616 9.66 [from part (b)]


ingredient weight (lbf) volume (ft³)

water (0.44)(611 lbf)= 269


269 lbf
= 4.31
lbf
62.4 
3
ft

air 0
3
3
ft
(0.04) (1 yd ) (27  )
3
yd

= 1.08

total 18.16

The volume of fine aggregate in 1 yd3 of the mix is

3
3
ft 3 3 3
(1 yd ) (27  ) − 18.16 ft = 8.84 ft (8.8 ft )
3
yd

Scenario
(f) What is the SSD weight of the sand in 1 yd3 of fresh concrete?

Answers
(A) 1250 lbf

(B) 1270 lbf

(C) 1300 lbf

(D) 1370 lbf

The answer is D.

Solution
(f) The weight of fine aggregate is

3
lbf
(8.84 ft ) (62.4  ) (2.48) = 1368 lbf  (1370 lbf)
3
ft

Scenario
(g) What is the absolute volume of the cement in 1 yd3 of fresh concrete?

Answers
(A) 3.1 ft3

(B) 3.2 ft3

(C) 3.3 ft3

(D) 3.4 ft3

The answer is A.
Solution
(g) Refer to part (e).

Scenario
(h) What is the volume of water designed for use in 1 yd3 of fresh concrete?

Answers
(A) 3.3 ft3

(B) 3.8 ft3

(C) 4.3 ft3

(D) 4.8 ft3

The answer is C.

Solution
(h) Refer to part (e).

Scenario
(i) If the oven-dry weight of the coarse aggregate were 1600 lbf, how much water would it need to absorb to
reach SSD conditions?

Answers
(A) 5.3 lbf

(B) 11 lbf

(C) 32 lbf

(D) 1100 lbf

The answer is B.

Solution
(i) The weight of the water is

(0.007) (1600 lbf) = 11.2 lbf (11 lbf)

Scenario
(j) The unit weight of this concrete mix is most nearly

Answers
(A) 140 lbf/ft3

(B) 143 lbf/ft3

(C) 145 lbf/ft3

(D) 150 lbf/ft3

The answer is B.
Solution
(j) The unit weight of the concrete is

lbf lbf lbf lbf


611  + 1616  + 269  + 1368 
3 3 3 3
yd yd yd yd

3
ft
27 
3
yd

3
= 143 lbf/ft
problem 3 in chapter 62

 Return to Questions (/admin/questions/0?sfield=magento_id&stext=0000055978&sdka=&stype=&sdiff=)

Test Bank Question


preview

QUESTION DATA
Question
Vendor
(Time limit: one hour) A tall column in an industrial storage building carries the vertical roof and lateral wind 0000055978
loads shown. The bottom end is built in, and the top end is fixed against translation, but free to rotate. The Flashcard
lateral wind loads are carried into a braced frame. The column is braced in its weak direction by a strut 24 ft 
from the bottom hinged end. There is no support of the column compression flange along its length. Make Solving Time
reasonable assumptions to simplify the structural analysis. (a) Evaluate a W24 × 104 section. Do not check
shear strength. (b) Verify that the lateral column deflection is acceptable. Difficulty
easy
Quantitative?
Yes
Wind=52 kips Status
997 Active
Created On
10/29/2018 06:26:33 PM
Published On
10/29/2018 06:26:33 PM
Modified On
863 08/29/2020 12:41:27 AM
OTHER VERSIONS

10/18/2018 04:58:11 PM
(/admin/questions/previ

672 10/29/2018 06:26:33 PM


(/admin/questions/previ

DISCIPLINES

PE Civil: Structural
(/admin/questions/index
Solution sfield=discipline&stext=
(a) Determine the effective length, Lx, of the column. The bottom is built in (fixed), and the top is free to Civil: Structural)
rotate, but fixed against translation. The deflection curvature will be the same as a built-in/pinned
KNOWLEDGE AREAS
combination. From Table 61.1, Kx = 0.80.
Component design and
Lx = Kx L = (0.80) (28 ft + 24 ft) = 41.6 ft
detailing
AISC Specification Chap. B, Sec. B2 defers to ASCE/SEI7 Chap. 2 for load factors. ASCE/SEI7 Sec. 2.4.1, (/admin/questions/index
combination 4, specifies the ASD load combination for roof live, dead, and wind loads. Particularly because sfield=area&stext=Com
of the large wind load component, other combinations result in smaller loadings. design and detailing)

PRODUCTS USED IN
P a = D + 0.75W + 0.75Lr [ASD]

ASCE/SEI7 Sec. 2.3.2, combination 4, specifies the LRFD load combination for roof live, dead, and wind
loads. Particularly because of the large wind load component, other combinations result in smaller loadings.

P u = 1.2D + 1.0W + 0.5Lr [LRFD]


For ASD,

P a = 17 kips + (0.75) (31 kips) + (0.75) (21 kips)

= 56 kips

For LRFD,

P u = (1.2) (17 kips) + (1.0) (31 kips) + (0.5) (21 kips)

= 61.9 kips

Since the bottom end is built in and the top end is free to rotate, this column behaves like a propped
cantilever in response to lateral loading. The difficulty in doing an exact analysis with the different wind
intensities, say by superposition, is that the locations of the points of the maximum moments are different for
the three different wind intensities. In order to incorporate the actual beam restraints without increasing the
workload significantly, simplify the loading. (If the column analysis shows marginal adequacy, a more exact
analysis can be performed.)

The various wind distribution magnitudes are not too dissimilar, so calculate an average wind loading.

lbf lbf
(15 ft) (403  ) + (25 ft) (518  )
ft ft

lbf
+ (12 ft) (598  )
ft
wave =
15 ft + 25 ft + 12 ft

  = 503.3 lbf/ft

There are no dead or live lateral loads. The most extreme case (of load factors) for wind only comes from
ASCE/SEI7, as before, but the load factors are not necessarily the same as for the vertical loading.

For ASD,

lbf
wa,lateral = 1.00wave = (1.00) (503.3  )
ft

  = 503.3 lbf/ft

For LRFD,

lbf
wu,lateral = 1.6wave = (1.6) (503.3  )
ft

  = 805.3 lbf/ft

Use App. 44.A, case 4.

For ASD,

lbf 2
(9) (503.3  ) (52 ft)
2
9wL ft
Mmax = =
128 lbf
(128) (1000  )
kip

  = 95.69 ft-kips

For LRFD,

lbf 2
(9) (805.3  ) (52 ft)
2
9wL ft
Mmax = =
128 lbf
(128) (1000  )
kip

  = 153.1 ft-kips
ASD Method Solution

P a = 56 kips

Mx = 95.69 ft-kips

My = 0 ft-kips

Determine the properties of a W24 × 104 for KL ≈ 42 ft. From AISC Manual Table 6-1,

−3 −1
1
p = 6.52 × 10 kips = 0.00652
kips

−3 −1
1
bx = 3.43 × 10 (ft-kips) = 0.00343
ft-kip

by = 0

Assume Pa/(Pn/Ωc) > 0.2. Then, from Eq. 62.10,

pP a + bx Mx + by My ≤ 1.0

1
= (0.00652 ) (56 kips)
kips

1
+ (0.00343 ) (95.69 ft-kips) + 0
ft-kip

= 0.693 ≤ 1.0 [adequate]

LRFD Method Solution

P u = 61.9 kips

Mx = 153.1 ft-kips

My = 0

Determine the properties of a W24 × 104 for KL ≈ 42 ft. From AISC Manual Table 6-1,

−3 −1
1
p = 4.33 × 10 kips = 0.00433 
kips

−3 −1
1
bx = 2.29 × 10 (ft-kips) = 0.00229
ft-kip

by = 0

Assume Pu/ϕPn > 0.2. Then, from Eq. 62.10,

pP u + bx Mx + by My ≤ 1.0

1
= (0.00433 ) (61.9 kips)
kips

1
+ (0.00229 ) (153.1 ft-kips) + 0
ft-kips

= 0.619 ≤ 1.0 [adequate]

(b) Use App. 44.A, case 4 with the average wind force. The maximum deflection is
Note that when calculating deflection due to wind, it is common to use serviceability wind
load of 0.6x0.7wave. This corresponds to 10-yr wind maps. The maximum deflection is
4
wave L
Δmax =
185EI
2
lbf 4
in
(503.3  ) (52 ft) (12  )
ft ft
  =

6
lbf 4
(185) (29 × 10   ) (3100 in )
2
in

  = 0.0319 ft

Δmax 0.0319 ft
= = 1/1630
L 52 ft

L/1630 is less than the traditional guideline that deflection should not exceed L/360 for architectural and
aesthetic reasons [AISC Commentary, Chap. L, Sec. L3]. The deflection is so small that it is not necessary
to perform a more exact deflection analysis.

A W24 × 104 beam is adequate for deflection.


problem 20 in chapter 17

 Return to Questions (/admin/questions/0?sfield=magento_id&stext=0000053256&sdka=&stype=&sdiff=)

Test Bank
Question preview

QUESTION DATA
Question
Vendor
The velocity of discharge from a fire hose is 50 ft/sec (15 m/s). The hose is oriented 45° from the 0000053256
horizontal. Disregarding air friction, the maximum range of the discharge is most nearly Solving Time

Answers Difficulty
(A) 45 ft (14 m) easy
Quantitative?
(B) 78 ft (23 m)
Yes
(C) 91 ft (27 m) Status
Active
(D) 110 ft (33 m)
Created On
10/29/2018 06:24:04
The answer is (B).
PM
Published On
Solution 10/29/2018 06:24:04
Customary U.S. Solution PM
Modified On
Use projectile equations. 08/29/2020 12:40:39
From Table 71.2, the maximum range of the discharge is given by AM
OTHER VERSIONS
⎛ ⎞
sin 2ϕ ft
2
sin (2) (45°) 10/18/2018 04:55:49 P
2 ⎜ ⎟
R = vo ( ) = (50  )  ⎜ ⎟
g sec

ft
⎟ (/admin/questions/prev
⎝ 32.2  ⎠
2
sec
10/29/2018 06:24:04 P
= 77.64 ft (78 ft)
(/admin/questions/prev
SI Solution DISCIPLINES
Use projectile equations.
PE Chemical
The maximum range of the discharge is given by (/admin/questions/inde
sfield=discipline&stext=
2
⎛ ⎞ Chemical)
sin 2ϕ m sin (2) (45°)
2
R = vo ( ) = (15  ) ⎜ ⎟
⎜ m ⎟
g s
⎝ 9.81  ⎠
s
2 PE Civil: Water
Resources and
= 22.94 m (23 m)
Environmental
(/admin/questions/inde
sfield=discipline&stext=
Civil: Water Resources
and Environmental)

KNOWLEDGE AREAS
problem 12 in chapter 35

 Return to Questions (/admin/questions/0?sfield=magento_id&stext=0000055653&sdka=&stype=&sdiff=)

Test Bank
Question preview

QUESTION DATA
Question
Vendor
(Time limit: one hour) A sandy clay soil was tested in a direct shear apparatus, with the following results. 0000055653
Solving Time

Difficulty
easy
Quantitative?
Yes
Status
Active
Created On
10/29/2018 06:25:03
PM
Published On
10/29/2018 06:25:03
PM
Modified On
08/29/2020 12:41:00
AM
Scenario OTHER VERSIONS

(a) What is the friction angle? 01/19/2018 11:04:44 P


(/admin/questions/prev
Answers
(A) 7° 10/29/2018 06:25:04 P
(/admin/questions/prev
(B) 14°
DISCIPLINES
(C) 23°

(D) 29° PE Civil: Geotechnical


(/admin/questions/inde
The answer is C. sfield=discipline&stext=
Civil: Geotechnical)
Solution KNOWLEDGE AREAS
(a) Plot the maximum shear stress from the stress-displacement curve versus the normal stress from
Soil Mechanics,
each individual test.
Laboratory Testing, and
Analysis
(/admin/questions/inde
sfield=area&stext=Soil
Mechanics, Laboratory
Testing, and Analysis)

PRODUCTS USED IN
The best fit line through the three data points represents the failure envelope for the soil. Measuring from
the plot,

ϕ = 23°

Scenario
(b) What is the cohesion intercept?

Answers
(A) 24 kPa

(B) 33 kPa

(C) 57 kPa

(D) 86 kPa

The answer is B.

Solution
The graph
(b) From in part
the graph (a) (a),
in part hasca=slope of tan(phi)=0.425. The cohesion intercept is the
33 kPa.
constant, c, of the linear equation representing the graph. Tao=0.425 sigma + c. Use any
of the data points to solve for c.

c = 33 kPa
problem 5 in chapter 59

 Return to Questions (/admin/questions/0?sfield=magento_id&stext=0000055955&sdka=&stype=&sdiff=)

Test Bank
Question preview

QUESTION DATA
Question
Vendor
(Time limit: one hour) The steel beam shown carries a live load of 1.4 kips/ft, excluding self-weight, over 0000055955
its entire span. Assume lateral bracing only at the reaction points. (Disregard the weight of the beam in the Solving Time
following questions.) Use either the ASD or LRFD method.
Difficulty
easy
Quantitative?
Yes
Status
Active
Created On
11/08/2018 07:41:36
PM
Published On
11/08/2018 07:41:36
Scenario PM
(a) The reaction (factored reaction, for LRFD) at A is most nearly Modified On
08/29/2020 12:41:24
Answers AM
OTHER VERSIONS
(A) 12 kips (24 kips)
03/21/2018 07:35:58 P
(B) 14 kips (28 kips)
(/admin/questions/prev
(C) 16 kips (32 kips)

(D) 18 kips (36 kips) 10/18/2018 04:58:04 P


(/admin/questions/prev
The answer is B.
10/29/2018 06:26:25 P
Solution (/admin/questions/prev

ASD Method Solution


11/08/2018 07:41:36 P
(a) Disregard the weight of the beam. (/admin/questions/prev
Taking clockwise moments as positive,
DISCIPLINES
∑ Mhinge to left = R A (20 ft)
PE Civil: Structural
kips 20 ft
− (1.4  ) (20 ft) ( ) = 0 (/admin/questions/inde
ft 2 sfield=discipline&stext=
RA = 14 kips
Civil: Structural)
LRFD Method Solution KNOWLEDGE AREAS
(a) Disregard the weight of the beam.
Component design and
kips detailing
wu = 1.6wL = (1.6) (1.4  ) = 2.24 kips/ft
ft (/admin/questions/inde
Taking clockwise moments as positive, sfield=area&stext=Com
design and detailing)
∑ Mhinge to left = R A (20 ft)

kips 20 ft PRODUCTS USED IN


− (2.24  ) (20 ft) ( ) = 0
ft 2

RA = 22.4 kips (24 kips)

Scenario
(b) The reaction at C is most nearly

Answers
(A) 36 kips (58 kips)

(B) 37 kips (60 kips)

(C) 39 kips (62 kips)

(D) 42 kips (67 kips)

The answer is C.

Solution
Customary U.S. Solution

(b) Disregard the weight of the beam.

∑ MD = (14 kips) (45 ft)

kips 45 ft
− (1.4  ) (45 ft) ( )
ft 2

kips 5 ft
+R C (20 ft) + (1.4  ) (5 ft) ( ) = 0
ft 2

RC = 38.5 kips (39 kips)

SI Solution

(b) Disregard the weight of the beam.

∑ MD = (22.4 kips) (45 ft)

kips 45 ft
− (2.24  ) (45 ft) ( )
ft 2

kips 5 ft
+R C (20 ft) + (2.24  ) (5 ft) ( ) = 0
ft 2

RC = 61.6 kips (62 kips)

Scenario
(c) The reaction at D is most nearly

Answers
(A) 13 kips (20 kips)

(B) 18 kips (28 kips)

(C) 19 kips (30 kips)


(D) 22 kips (35 kips)

The answer is B.

Solution
Customary U.S. Solution

(c) The reaction at D is

kips
R D = (1.4  ) (50 ft) − 14 kips − 38.5 kips
ft

= 17.5 kips (18 kips)

SI Solution

(c) The reaction at D is

kips
R D = (2.24  ) (50 ft) − 22.4 kips − 61.6 kips
ft

= 28 kips

Scenario
(d) The absolute maximum value of shear (factored shear, for LRFD) is most nearly

Answers
(A) 12 kips (19 kips)

(B) 14 kips (22 kips)

(C) 18 kips (29 kips)

(D) 21 kips (34 kips)

The answer is D.

Solution
Customary U.S. Solution

(d) Draw the shear and moment diagram.


From the shear diagram, the absolute maximum shear is 21 kips.

SI Solution

(d) Draw the shear and moment diagram.


From the shear diagram, the absolute maximum shear is 33.6 kips  (34 kips) .

Scenario
(e) The absolute maximum shear occurs closest to

Answers
(A) support A

(B) hinge B

(C) support C

(D) support D

The answer is C.

Solution
Customary U.S. Solution

(e) From the shear diagram, the absolute maximum shear occurs just to the left of support C.

SI Solution

(e) From the shear diagram, the absolute maximum occurs just to the left of support C.

Scenario
(f) The absolute maximum moment is most nearly

Answers
(A) 70 ft-kips (140 ft-kips)

(B) 79 ft-kips (158 ft-kips)

(C) 88 ft-kips (176 ft-kips)

(D) 98 ft-kips (196 ft-kips)

The answer is C.

Solution
Customary U.S. Solution

(f) From the moment diagram, the absolute maximum moment is 87.5 ft-kips (88 ft-kips).

SI Solution

(f) From the moment diagram, the absolute maximum moment is 140 ft-kips.

Scenario
(g) The absolute maximum moment occurs

Answers
(A) halfway between A and B
(B) at support C

(C) halfway between C and D

(D) at support D

The answer is B.

Solution
Customary U.S. Solution

(g) From the moment diagram, the absolute maximum moment occurs at support C.

SI Solution

(g) From the moment diagram, the absolute maximum moment occurs at support C.

Scenario
(h) If the unbraced length is 25 ft, what is the lightest W18 section of A992 steel that can be used, based
on moment-resisting capacity?

Answers
(A) W18 × 55

(B) W18 × 60

(C) W18 × 65

(D) W18 × 76

The answer is D.

Solution
Customary U.S. Solution

(h) Lb = 25 ft; Mmax = 87.5 ft-kips.

From AISC Manual Table 3-10, select a W18 × 76.

The allowable moment, Mn/Ω, of 272 ft-kips far exceeds the actual maximum moment of 87.5 ft-kips
(excluding beam weight). There is ample reserve capacity for the additional moment due to the weight of
the beam.

SI Solution

(h) Lr = 25 ft; Mmax = 140 ft-kips.

From AISC Manual Table 3-10, select a W18 × 76.

The design moment, ϕMn, of 410 ft-kips far exceeds the actual maximum (or required) moment of 140 ft-
kips (excluding beam weight). There is ample reserve capacity for the additional moment due to the
weight of the beam.

Scenario
(i) For the lightest beam chosen in part (h), the actual maximum shear stress (ignoring the weight of the
beam) is most nearly
Answers
(A) 2.0 ksi (3.2 ksi)

(B) 2.4 ksi (3.8 ksi)

(C) 2.7 ksi (4.3 ksi)

(D) 3.4 ksi (5.4 ksi)

The answer is C.

Solution
Customary U.S. Solution

(i) For a W18 × 76,

d = 18.2 in

tw = 0.425 in

The actual shear stress is

V V 21 kips
fv = = =
Aw dtw (18.2 in) (0.425 in)

2
= 2.715 kips/in (2.7 ksi)

SI Solution

(i) For a W18 × 76 ,

d = 18.2 in

tw = 0.425 in

The actual shear stress is

Vu 33 kips
fv = =
dtw (18.2 in) (0.425 in)

2
= 4.27 kips/in (4.3 ksi)

Scenario
(j) If the beam chosen in part (h) is constructed of 42 ksi steel, the maximum unbraced length that permits
the development of a flexural strength of Mp is most nearly

Answers
(A) 60 in

(B) 96 in

(C) 121 in

(D) 240 in

The answer is C.

Solution
Customary U.S. Solution

(j) In order to develop Mp, the unbraced length mustbe less than Lp. From AISC Manual Part 1, for aW18 ×
76, ry =  2.61 in.

From Eq. 59.6,


−−−−− −−−−− −

kips

−−
−  29,000 
2
E  in
Lp = 1.76r y √ = (1.76) (2.61 in) 
Fy  kips
⎷ 42 
2
in

= 120.7 in (121 in)

SI Solution

(j) In order to develop Mp, the unbraced length mustbe less than Lp. From AISC Manual Part 1, for aW18 ×
76, ry =  2.61 in.

From Eq. 59.6,


−−−−−−−−− −

kips

−−
−  29,000  2
E  in
Lp = 1.76r y √ = (1.76) (2.61 in) 
Fy  kips
⎷ 42 
2
in

  = 120.7 in (121 in)
problem 1 of chapter 36

 Return to Questions (/admin/questions/0?sfield=magento_id&stext=0000055646&sdka=&stype=&sdiff=)

Test Bank
Question preview

QUESTION DATA
Question
Vendor
A mat foundation is to be used to support a building with dimensions of 80 ft × 40 ft and a 5200 ton total 0000055646
load. The mat is located 8 ft below the ground surface. The soil beneath the mat is a sand with a specific Solving Time
weight of 120 lbf/ft3 and an average SPT N-value of 18.
Difficulty
easy
Scenario Quantitative?
Yes
(a) What is the allowable bearing capacity of the mat?
Status
Active
Answers
Created On
(A) 2.2 tons/ft2 04/14/2020 11:19:45
AM
(B) 2.5 tons/ft2
Published On
(C) 2.9 tons/ft2 04/14/2020 11:19:45
AM
(D) 5.4 tons/ft2
Modified On
The answer is B. 08/29/2020 12:41:01
AM
OTHER VERSIONS
Solution
(a) The SPT N-value should be corrected for the overburden pressure. At the base of the mat foundation, 03/21/2018 07:35:29 P
the overburden pressure is (/admin/questions/prev

lbf
(120  ) (8 ft) 10/29/2018 06:25:06 P
ft
3
(/admin/questions/prev
poverburden = γDf =
lbf
2000 
ton 04/14/2020 11:19:45 A
= 0.48 tons/ft
2
(/admin/questions/prev

From Table 36.6, the correction factor is Cn ≈ 1.21. DISCIPLINES

At a depth of Df + B below the ground surface, the overburden pressure is PE Civil: Geotechnical
(/admin/questions/inde
lbf
(120  ) (8 ft + 40 ft) sfield=discipline&stext=
3

poverburden = γ (Df + B) =
ft Civil: Geotechnical)
lbf
2000 
ton PE Civil: Construction
2
= 2.88 tons/ft (/admin/questions/inde
sfield=discipline&stext=
From Table 36.6, the correction factor is Cn ≈ 0.63. This is the factor that should be used for design, since
Civil: Construction)
it will result in the lowest N-value.

PE Civil: Structural
(/admin/questions/inde
The net allowable bearing capacity for sand is given in Eq. 36.23. The correction factor Cw from Eq. 36.13 sfield=discipline&stext=
is assumed equal to 1.0 since there is no mention of groundwater being within the influence zone of the Civil: Structural)
mat.

qnet, a = 0.22C wC nN = (0.22) (1) (0.63) (18)


PE Civil: Transportatio
(/admin/questions/inde
2 2
= 2.49 tons/ft (2.5 tons/ft ) sfield=discipline&stext=
Civil: Transportation)

Scenario PE Civil: Water


(b) What is the factor of safety against bearing capacity failure? Resources and
Environmental
Answers (/admin/questions/inde
sfield=discipline&stext=
(A) 1.3 Civil: Water Resources
(B) 1.9 and Environmental)

(C) 2.2 KNOWLEDGE AREAS

(D) 4.4 Shallow Foundations


(ASD or LRFD)
The answer is D. (/admin/questions/inde
sfield=area&stext=Sha
Solution Foundations (ASD or
LRFD))
(b) By Eq. 36.24,

P total
pnet,actual = − γD f
Soil Mechanics
A raft (/admin/questions/inde
lbf sfield=area&stext=Soil
(120  ) (8 ft)
5200 tons
3 Mechanics)
ft
= −
(80 ft) (40 ft) lbf PRODUCTS USED IN
2000 
ton
2
= 1.15 tons/ft

The factor of safety against bearing capacity failure based on allowable stress is given by Eq. 36.4.

tons
2.49 
qnet,a 2
ft
=
pnet,actual tons
1.15 
2
ft

= 2.2

Since a factor of safety of 2 is incorporated into the determination of qnet, as discussed in Eq. 36.12

F = (2) (2.2) = 4.4

You might also like